limit of integral $n\int_{0}^{1} x^n f(x) \text{d}x$ as $n\rightarrow \infty$

I am trying to solve the following problem at the level of a senior undergrad analysis level. So, the problem is as follows: We are given a function $f$ which is continuous on the interval $\left [ 0,1 \right ]$, and the question is to find the limit: $$\lim_{n\rightarrow \infty}\int_{0}^{1}x^{n}f(x)dx\;.$$ The second part of the problem is to deduce the following limit: $$\lim_{n\rightarrow \infty}n\int_{0}^{1}x^{n}f(x)dx\;.$$

For the first part: I just did the following: For every $0\leq x< 1$: $x\leq M$, where $0< M< 1$. Then: $$\int_{0}^{1}x^{n}f(x)dx\leq M^{n}\int_{0}^{1}f(x)dx\;.$$ Then: $$\lim_{n\rightarrow \infty }\int_{0}^{1}x^{n}f(x)dx\leq \lim_{n\rightarrow \infty }M^{n}\int_{0}^{1}f(x)dx= 0.\int_{0}^{1}f(x)dx=0\;,$$ so $$\lim_{n\rightarrow \infty }\int_{0}^{1}x^{n}f(x)dx=\lim_{n\rightarrow \infty }f(1)\int_{0}^{1}1dx=f(1)\;.$$ Does that make sense? If not, please show me the correct one.

As for the second part, I have no idea what to do. Any help?


For the first part, use the fact that a continuous function on $[0,1]$ is bounded, say $|f(x)|\le M$. Then $$ \Bigg|\int_0^1 x^nf(x)\,dx\Bigg|\le\int_0^1 Mx^n\,dx =\frac{M}{n+1}\to 0 \quad\text{as}\quad n\to\infty. $$

For the second part $\dots$ $$ n\int_0^1 x^nf(x)\,dx = n\int_0^1 x^n\Big(f(x)-f(1)\Big)\,dx + \frac{n}{n+1}f(1). $$ The last term tends to $f(1)$ as $n\to\infty$. To show that the first term on the right tends to zero, suppose $\varepsilon>0$. Choose $a<1$ so that $|f(x)-f(1)|<\varepsilon$ for $a<x<1$. Then $$ \Bigg|\,n\int_0^1 x^n\Big(f(x)-f(1)\Big)\,dx\Bigg| \le \,n\int_0^a x^n\Big|f(x)-f(1)\Big|\,dx+ \,n\int_a^1 x^n\Big|f(x)-f(1)\Big|\,dx\\ \le 2M\frac{n}{n+1}a^n + \frac{n}{n+1}\varepsilon $$ and this tends to $\varepsilon$ as $n\to\infty$.


As to the second question: The integral can easily be computed explicitly when $f$ is a polynomial, and the limit then calculated. Because the integral $n\int_0^1 x^n dx$ is bounded above by $1$, the Weierstrass approximation theorem can be applied to deal with general continuous $f$.


For second part, we can find limit using little functional analysis. Let denote with $A_n:C[0,1] \to \mathbb{C}$ linear operator $A_n(f)=n\displaystyle\int_0^1 x^n f(x)\, dx$. We see that

$$\| A_n f \|_{\mathbb{C}} = |A_n (f)|=\left\lvert n\int_0^1 x^n f(x)\, dx\right\rvert \leqslant n \|f\|_{\infty} \int_0^1 x^n \, dx=\frac{n}{n+1}\| f\|_{\infty} < \|f\|_{\infty},$$

so $\sup\limits_{n \in \mathbb{N}} \|A_n \| < +\infty$.

Now, we are going to check our operator on fundamental set $\{x \mapsto x^k \mid k \in \mathbb{N}_0\}$ (because $\overline{\operatorname{span}\{x \mapsto x^k \mid k \in \mathbb{N}_0\}}=C[0,1]$):

$$\lim_{n\to \infty}A_n(x^k)=\lim_{n\to \infty} n \int_0^1 x^n x^k\, dx=\lim_{n \to \infty} \frac{n}{n+k+1}=1=x^k(1).$$

Banach–Steinhaus theorem gives us that limit exist and

$$\lim_{n \to \infty} A_n(f)=f(1).$$